Difference between revisions of "2021 IMO Problems/Problem 6"

(Problem)
 
(3 intermediate revisions by 2 users not shown)
Line 1: Line 1:
==Problem==
+
== Problem ==
Let <math>m\ge 2</math> be an integer, <math>A</math> be a finite set of (not necessarily positive) integers, and <math>B_1,B_2,B_3,...,B_m</math> be subsets of <math>A</math>. Assume that for each <math>k = 1, 2,...,m</math> the sum of the elements of <math>B_k</math> is <math>m^k</math>. Prove that <math>A</math> contains at least <math>m/2</math> elements.
+
Let <math>m \ge 2</math> be an integer, <math>A</math> be a finite set of (not necessarily positive) integers, and <math>B_1, B_2, B_3 , \ldots, B_m</math> be subsets of <math>A</math>. Assume that for each <math>k = 1, 2,...,m</math> the sum of the elements of <math>B_k</math> is <math>m^k</math>. Prove that <math>A</math> contains at least <math>m/2</math> elements.
  
==Video solution==
+
== Video solution ==
https://www.youtube.com/watch?v=vUftJHRaNx8 [Video contains solutions to all day 2 problems]
+
https://youtu.be/vUftJHRaNx8 [Video contains solutions to all day 2 problems]
 +
 
 +
== See also ==
 +
{{IMO box|year=2021|num-b=5|after=Last Problem}}
 +
 
 +
[[Category:Olympiad Algebra Problems]]

Latest revision as of 10:44, 18 June 2023

Problem

Let $m \ge 2$ be an integer, $A$ be a finite set of (not necessarily positive) integers, and $B_1, B_2, B_3 , \ldots, B_m$ be subsets of $A$. Assume that for each $k = 1, 2,...,m$ the sum of the elements of $B_k$ is $m^k$. Prove that $A$ contains at least $m/2$ elements.

Video solution

https://youtu.be/vUftJHRaNx8 [Video contains solutions to all day 2 problems]

See also

2021 IMO (Problems) • Resources
Preceded by
Problem 5
1 2 3 4 5 6 Followed by
Last Problem
All IMO Problems and Solutions